Đến nội dung

hoangson2598

hoangson2598

Đăng ký: 18-10-2013
Offline Đăng nhập: 30-07-2016 - 23:03
****-

#642284 Cmr:$3(\frac{1}{\sqrt{x}}+\...

Gửi bởi hoangson2598 trong 26-06-2016 - 16:57

Cho a,b,c là các số thực dương thỏa mãn ab+bc+ca+abc=4.Cmr:

$3(\frac{1}{\sqrt{x}}+\frac{1}{\sqrt{y}}+\frac{1}{\sqrt{z}})^2\geq (x+2)(y+2)(z+2)$

Đề Bắc Giang à

a10411.png




#641756 $\left\{\begin{matrix} x^2+(y^2-y+1)\...

Gửi bởi hoangson2598 trong 22-06-2016 - 17:13

$\left\{\begin{matrix} x^2+(y^2-y+1)\sqrt{x^2+2}-y^3+y+2=0 & \\ \sqrt[3]{y^2-3}-\sqrt{xy^2-2x-2}+x=0 & \end{matrix}\right.$

 

Nếu phương trình (1) viết lại như sau: $x^2+(y^2-y+1)\sqrt{x^2+2}-y^3-y+2=0$

Đặt: $a=\sqrt{x^2+2},a>0$

Viết lại phương trình (1): $(y-a)[y^2+2(a+1)]=0$

Do điều kiện nên: $y^2+2(a+1)>0$

Suy ra: $y^2-x^2=2$

Từ đó thế vào (2) ta giải phương trình: $\sqrt[3]{x^2-1}+x=\sqrt{x^3-2}$

Ta được x=3.

Do y>0 Nên $y=\sqrt{11}$

Chắc có chút nhầm lẫn với đề chuyên Hà Tĩnh


  • NAT yêu thích


#639660 CMR: $\sum \frac{1}{a} +\frac{9...

Gửi bởi hoangson2598 trong 11-06-2016 - 20:51

Cho a,b,c là độ dài 3 cạnh tam giác. CMR:

$\sum \frac{1}{a} +\frac{9}{a+b+c} \geq 4\sum \frac{1}{a+b}$

Nhìn bạn bên trên giải mà sợ quá.

Bài này quy đồng lên là xong chứ sao.

Nhân 2 vế với (a+b+c) ta được:

$3+\sum \frac{a}{b}+\sum \frac{a}{c}+9\geq 4(3+\sum \frac{a}{b+c})\Leftrightarrow \sum (\frac{a}{b}+\frac{a}{c})\geq \sum \frac{4a}{b+c}$ (luôn đúng theo cosi)




#638827 Tổng hợp các bài BĐT trong các đề thi thử THPT Quốc Gia môn Toán năm 2017

Gửi bởi hoangson2598 trong 07-06-2016 - 23:40

Bài 141:

Cho $0\leq a\leq b\leq 1\leq c$ và  $2b^2+c^2+4(2a+b+c)=18$

Tìm max $P=ab^2+bc^2+ca^2-\frac{13}{2a-5b+6(\sqrt{b}+\sqrt[3]{4bc})}$




#638613 làm thế nào để tập chung nhất khi đọc sách, tài liệu toán

Gửi bởi hoangson2598 trong 06-06-2016 - 22:29

Chỉ cần sách đủ hay để mình thích đọc là sẽ tập trung. Khi đấy càng đọc sẽ càng tập trung. Trừ khi mình không cảm nhận được cái hay của nó thôi




#637915 $\large a\sqrt{b+c}+b\sqrt{a+c}+c...

Gửi bởi hoangson2598 trong 03-06-2016 - 22:05

CHo a;b;c là các số thực dương thỏa mãn: $ab+bc+ac=1$

CMR                 $\large a\sqrt{b+c}+b\sqrt{a+c}+c\sqrt{a+b}\leq \sqrt{2(a+b+c)}$

Áp dụng bunhia:

$VP=\sum \sqrt{a}\sqrt{ab+ac}\leqslant \sqrt{(a+b+c)(2ab+2ac+2bc)}=\sqrt{2(a+b+c)}=VT$




#630624 Tổng hợp các bài BĐT trong các đề thi thử THPT Quốc Gia môn Toán năm 2017

Gửi bởi hoangson2598 trong 01-05-2016 - 20:43

Bài 51

Cho $\frac{1}{3}< x\leq \frac{1}{2}$ và $y\geq 1$

Tìm giá trị nhỏ nhất:

$P=x^2+y^2+\frac{x^2y^2}{((4x-1)y-x)^2}$




#627122 $\frac{1}{\sqrt{2x^2+(3-\sqrt{3...

Gửi bởi hoangson2598 trong 14-04-2016 - 20:07

Tìm GTNN của

$P=\frac{\sqrt{3(2x^2+2x+1)}}{3}+\frac{1}{\sqrt{2x^2+(3-\sqrt{3})x+3}}+\frac{1}{\sqrt{2x^2+(3+\sqrt{3})x+3}}$




#626003 $\frac{(a+b+c)^3}{abc}+(\frac{ab+ac+b...

Gửi bởi hoangson2598 trong 08-04-2016 - 22:17

Em nghĩ đề là : 

Cho a, b, c > 0. Chứng minh rằng  :

$\frac{(a+b+c)^{3}}{abc}+(\frac{a^{2}+b^{2}+c^{2}}{ab+bc+ca})^{2}\geq 28$

Giải (theo đề sửa) : 

_ Áp dụng Bất đẳng thức AM-GM, ta có : 

$a+b+c\geq 3\sqrt[3]{abc}\Leftrightarrow \frac{(a+b+c)^{3}}{27}\geq abc\Leftrightarrow \frac{1}{abc}\geq \frac{27}{(a+b+c)^{3}}\Leftrightarrow \frac{(a+b+c)^{3}}{abc}\geq 27$

_ Có đánh giá quen thuộc : 

$a^{2}+b^{2}+c^{2}\geq ab+bc+ca\Leftrightarrow \frac{1}{ab+bc+ca}\geq \frac{1}{a^{2}+b^{2}+c^{2}}\Leftrightarrow \frac{a^{2}+b^{2}+c^{2}}{ab+bc+ca}\geq 1$

_ Bình phương BĐT 2 lên rồi cộng lại, ta có điều phải chứng minh.

_ Dấu "=" khi : $a=b=c$

Đề đúng rồi đấy, không đơn giản dùng cosi ra luôn được đâu.




#616244 Chứng minh \[\left( {1 + \frac{1}{{xy...

Gửi bởi hoangson2598 trong 21-02-2016 - 16:23

$(x-1)(y-1) \ge 0 \Rightarrow xy + 1 \ge x+y $
Tương tự $ yz + 1 \ge y+z $ và $ zx + 1 \ge z+x $
Cộng các vế vào ta được $ xy+yz+xz +3 \ge 2(x+y+z) $
Vì $ xyz \leq 1 $ nên $ 2(x+y+z) \ge ( 1+xyz)(x+y+z) $
Do đó : $xy+yz+xz +3 \ge (1+xyz)(x+y+z) $ $ \Leftrightarrow \[\left( {1 + \frac{1}{{xyz}}} \right)\left( {x + y + z} \right) \leq 3 + \frac{1}{x} + \frac{1}{y} + \frac{1}{z}\] $

 

Chia hai vế cho xyz đoạn cuối thì $\frac{3}{xyz}\geq 3$ mà. Không suy được.




#613672 Tìm giá trị lớn nhất, nhỏ nhất của biểu thức A=x+y+1, biết $x^{2...

Gửi bởi hoangson2598 trong 08-02-2016 - 19:53

Tìm giá trị lớn nhất, nhỏ nhất của biểu thức A=x+y+1, biết $x^{2} + 2xy + 7(x+y) + 2y^{2} + 10 = 0$

Từ giả thiết suy ra: 

$(x+y)^2+7(x+y)+10=-y^2\leq 0\Leftrightarrow -5\leq x+y\leq -2\Leftrightarrow -4\leq A\leq -1$

Kết luận $MinA=-4$ khi $x=5$, $y=0$

              $MaxA= -1$ khi $x=-2$, $y=0$




#606127 Giải Hệ Phương Trình

Gửi bởi hoangson2598 trong 30-12-2015 - 15:53

bất đẳng thức sai rồi bạn 

Áp dụng bunhia: 

$(mn+1)(\frac{m}{n}+1)\geq (m+1)^2\Leftrightarrow \frac{1}{(m+1)^2}\geq \frac{1}{mn+1}(\frac{n}{m+n})$

Tương tự ta có $\frac{1}{(n+1)^2}\geq \frac{1}{mn+1}(\frac{m}{m+n})$

Cộng vào suy ra 

$\frac{1}{(m+1)^2}+\frac{1}{(n+1)^2}\geq \frac{1}{mn+1}$




#606064 Giải Hệ Phương Trình

Gửi bởi hoangson2598 trong 30-12-2015 - 00:12

giải hpt: 

 

$\frac{2}{(\sqrt{x}+\sqrt{y})^{2}}+\frac{1}{x+\sqrt{y(2x-y)}}=\frac{2}{y+\sqrt{X(2x-y)}}$ 

$2(y-4)\sqrt{2x-y-3}-(x-6)\sqrt{x+y+1}=3(y-2)$

chia cả hai mẫu của phương trình 1 cho y rồi đặt $\frac{x}{y}=a$ ta được

$\frac{1}{(\sqrt{a}+1)^2}+\frac{1}{(\sqrt{2a-1}+1)^2}=\frac{1}{1+\sqrt{a(2a-1)}}$

Đặt $\sqrt{a}=m$, $\sqrt{2a-1}=n$ ta được:

$\frac{1}{(m+1)^2}+\frac{1}{(n+1)^2}=\frac{1}{1+mn}$ 

Một bđt quen thuộc suy ra 

$m=n\Leftrightarrow a=2a-1\Leftrightarrow a=1\Leftrightarrow x=y$




#603893 $P=x^2+y^2+z^2$

Gửi bởi hoangson2598 trong 18-12-2015 - 23:55

 

 

Từ giả thiết$=>\frac{(x+y+z)^3-1}{3(x+y+z)}=xy+yz+zx$

$=>P=(x+y+z)^2-\frac{2(x+y+z)^3-2}{3(x+y+z)}=t^2+\frac{2t^3-2}{3t}$

Ta có: $P'(t)=\frac{2(t^3-1)}{2t^2}=0<=>t=1$

$=>P\geqslant 1$

 

Từ giả thiết$=>\frac{(x+y+z)^3-1}{3(x+y+z)}=xy+yz+zx$

$=>P=(x+y+z)^2-\frac{2(x+y+z)^3-2}{3(x+y+z)}=t^2+\frac{2t^3-2}{3t}$

Ta có: $P'(t)=\frac{2(t^3-1)}{2t^2}=0<=>t=1$

$=>P\geqslant 1$

Nhầm một chút nhỉ!




#603017 $\frac{2z^2+zx}{(x+\sqrt{yz}+y)^2...

Gửi bởi hoangson2598 trong 13-12-2015 - 19:11

Cho x, y, z là các số thực dương. Chứng minh rằng:

$\frac{2x^2+xy}{(y+\sqrt{zx}+z)^2}+\frac{2y^2+yz}{(z+\sqrt{xy}+x)^2}+\frac{2z^2+zx}{(x+\sqrt{yz}+y)^2}\geq 1$